Answered step by step
Verified Expert Solution
Link Copied!

Question

1 Approved Answer

Question 26 (3 points) The company produces two different models having different profit margin. Based on the information below, compute total sales mix variance (in

image text in transcribedimage text in transcribedimage text in transcribedimage text in transcribedimage text in transcribed

Question 26 (3 points) The company produces two different models having different profit margin. Based on the information below, compute total sales mix variance (in terms of CM) of the company. Budget Actual VC per VC per Model Selling Price Units Sold Selling Price Units Sold Unit Unit Model A $120 $60 1,500 $100 $50 1,170 Model B $140 $100 1,500 $150 $110 1,430 Actual Units Sold for all models Actual Budgeted Sales-Mix - Percentage Sales-Mix ) x Budgeted Contribution Margin per Unit Sales- Mix Variance ( Percentage For your convenience, the UCM and sales mix for each product are computed in the following table: Budget Actual UCM Sales mix UCM Sales mix $60 50% $50 45% $40 50% $40 55% a) $2,600 U b) $3,700 U c) $3,700 F d) $2,600 F Question 27 (3 points) Using the same data in Question 26, compute total sales quantity variance of the company. Actual Total units Sold - Budgeted Sales- Budgeted Sales- Budgeted Total Units Sold x Mix Percentages X Contribution Margin = Quantity per Unit Variance a) $20,000 u b) $22,500 U c) $22,500 F d) $20,000 F Question 28 (3 points) Using the same data in Question 26, the budgeted market share based on total sales of 37,500 units was 8% (for 3,000 units) when actual total sale volume was 26,000 units in the market. Assuming the budgeted contribution margin per composite unit was $100, compute the market share variance. Market- share Variance Actual Market size in Units xl Actual market share Budgeted Budgeted Contribution market ) ~ Margin per composite unit share for budgeted mix a) $52,000 b) $43,000 U c) $52,000 F d) $43,000 F Question 29 (3 points) Using the same data in Question 26, compute the market size variance. Assume that the budgeted contribution margin per composite unit was $100. Market- size Variance = ( Actual Market size Budgeted Budgeted Budgeted Contribution market ) market X Margin per composite unit share for budgeted mix size O a) $92,000 F b) $68,000 F c) $92,000 u d) $68,000 U

Step by Step Solution

There are 3 Steps involved in it

Step: 1

blur-text-image

Get Instant Access to Expert-Tailored Solutions

See step-by-step solutions with expert insights and AI powered tools for academic success

Step: 2

blur-text-image

Step: 3

blur-text-image

Ace Your Homework with AI

Get the answers you need in no time with our AI-driven, step-by-step assistance

Get Started

Recommended Textbook for

Loose Leaf For Financial Accounting Fundamentals

Authors: John Wild, Ken Shaw, Barbara Chiappetta

6th Edition

1260151980, 978-1260151985

More Books

Students also viewed these Accounting questions

Question

Examine various types of executive compensation plans.

Answered: 1 week ago

Question

1. What is the meaning and definition of banks ?

Answered: 1 week ago

Question

2. What is the meaning and definition of Banking?

Answered: 1 week ago

Question

3.What are the Importance / Role of Bank in Business?

Answered: 1 week ago